Difference between revisions of "Aufgaben:Problem 5"

From Ferienserie MMP2
Jump to: navigation, search
m (nitpicks)
 
(21 intermediate revisions by 7 users not shown)
Line 1: Line 1:
=Problem=
+
Follow the link! (Original Solution Sheldon)
Show, by using the Fourier series, that
+
[https://polybox.ethz.ch/public.php?service=files&t=3cd27159ac0fca5b53c53fe4cff98dcc]
$$\sum_{k=1}^\infty \frac{1}{k^n} = - \frac{(2 \pi i)^n}{2(n!)} B_n,\ n \in 2 \mathbb{Z}_{>0}$$
+
where \(B_n\) are the Bernoulli numbers defined as
+
$$\frac{z}{e^z - 1} = \sum_{n=0}^\infty B_n \frac{z^n}{n!}$$
+
  
''Hint:'' consider \( B_n(x) := \sum_{k \in \mathbb{Z} \setminus \{0\}} k^{-n} e^{ikx},\ n \in \mathbb{Z}_{\geq 2},\ x \in \mathbb{R} \)
+
An alternative solution can be found here (3.). (MuLö Linalg)
 +
[http://www.math.ethz.ch/education/bachelor/lectures/hs2013/math/linalg1/l12]
  
 +
== Task ==
 +
Determine the center of the algebra of complex d × d-matrices.
 +
Hint: the center of an algebra \(A\) is defined as \(Z(A) = \{\phi \in A | \phi \psi = \psi \phi \; \forall \psi \in A \} \).
  
=Solution Sketch=
+
== Problem 5 (Sheldon) ==
The solution below is extremely comprehensive, if not expletive, because I felt that the [http://www.math.ethz.ch/~gruppe5/group5/group5.php?lectures/mmp/hs11/ml03.pdf Musterlösung] of the MMP Series that treated the problem was lacking some fairly important steps/guidelines. Therefore find a '''TL;DR''' here:
+
Let big bold letters denote elements of \(\mathbb{C}^{d \times d}\) and small Greek letters complex numbers.
  
# See that \(B_n(0) = 2 \sum_{k=1}^\infty k^{-n}\). We now want to find the \(B_n(0)\) for \(n \in 2\mathbb{Z}_{>0}\)
+
Let \(M \in Z\)
# We find a polynomial with Fourier coefficients \(c_0=0\) and \(c_k = \frac{1}{k^2}\). <br/> On \([0,2\pi]\), this polynomial will thus be equal to \(B_2(x)\)
+
# \(B'_n(x) = iB_{n-1}(x)\), therefore all \(B_n(x)\) will be polynomials, too. We find a recursion relation for \(B_n(x)\)
+
# Using the \(2\pi\)-periodicity of \(B_n(x)\) (for all \(n \geq 2\)), we find a recursion relation for the \(B_n(0)\)
+
# We prove the postulated identity by relating the definition for the \(B_n\) given in the problem to the recursion for the \(B_n(0)\) found above.
+
  
 +
We make the following observations and conclude that \( Z = \{\lambda\mathbb{I}\}\)
  
=Solution=
 
As suggested in the problem, we will have a look at the convergent fourier series
 
$$B_n(x) := \sum_{k \in \mathbb{Z} \setminus \{0\}} k^{-n} e^{ikx},\ n \in \mathbb{Z}_{\geq 2},\ x \in \mathbb{R}$$
 
with Fourier coefficients
 
$$c_k = \begin{cases}
 
0 & \text{for } k = 0\\
 
\frac{1}{k^n} & \text{for } k \neq 0
 
\end{cases}$$
 
  
 +
Obviously, the elementes of the center are closed under scalar multiplication (they even form a vector space over \(\mathbb{C}\)). \(\mathbb{I} \in Z \Rightarrow \{\lambda\mathbb{I}\} \subseteq Z\)
  
It is obvious that \( B_n(x) \) is \(2\pi\)-periodic and, by differentiating every term separately, that
 
$$B'_n(x) = iB_{n-1}(x)$$
 
  
Also, for \(n \in 2\mathbb{Z}_{>0}\), we see that
+
The theorem about Jordan decomposition states that \(M\) is conjugate to a matrix in Jordan normal form \(J\) and \(J = CMC^{-1}=MCC^{-1}=M\).
$$B_n(0) = \sum_{k \in \mathbb{Z} \setminus \{0\}} k^{-n} e^{ik\,0}
+
= \sum_{k=-\infty}^{-1} k^{-n} + \sum_{k=1}^\infty k^{-n}
+
= 2 \sum_{k=1}^\infty k^{-n}
+
\tag{$\ddagger$}$$
+
where the last step follows from the fact that \(n\) is even.
+
  
<hr style="border:0;border-top:thin dashed #ccc;background:none;"/>
+
\(M\) is a diagonal matrix.
  
We will first show that \( B_2(x) = f(x) := \pi^2/3 - \pi x + x^2/2 \) for  \( x \in [0, 2\pi]\).
+
Suppose to the contrary that \(M\) has a '1' in the superdiagonal, then \(2M\) is supposed to be in the center as well, but it can't, because it is not in Jordan normal form.
  
Recall the definition of the Fourier coefficients for \(2\pi\)-periodic functions in \(C^\infty(\mathbb{R})\):
 
$$c_k = \frac{1}{2\pi} \int_0^{2\pi} f(x) \: e^{-ikx} \: dx$$
 
  
We can easily verify the following identities:
+
All diagonal entries of \(M\) are equal.
$$\begin{align}
+
\frac{1}{2\pi} \int_0^{2\pi} e^{-ikx} \: dx &\ =\
+
\begin{cases}
+
1 & \text{for } k = 0\\
+
0 & \text{for } k \neq 0
+
\end{cases} \\
+
\frac{1}{2\pi} \int_0^{2\pi} x \: e^{-ikx} \: dx &\ =\
+
\begin{cases}
+
\pi & \text{for } k = 0\\
+
\frac{i}{k} & \text{for } k \neq 0
+
\end{cases} \\
+
\frac{1}{2\pi} \int_0^{2\pi} x^2 \: e^{-ikx} \: dx &\ =\
+
\begin{cases}
+
\frac{4}{3} \pi^2 & \text{for } k = 0\\
+
\frac{2(1+ik\pi)}{k^2} & \text{for } k \neq 0
+
\end{cases}
+
\end{align}$$
+
  
Calculating the Fourier coefficients for \(f(x)\) then yields
+
Let \( M =\begin{pmatrix} \lambda_1 & 0 & \ldots & 0 \\ 0 & \lambda_2 & \ldots & 0 \\ \vdots & &\ddots& \\ 0 & \ldots & 0 & \lambda_d \end{pmatrix} \) and \( E_{ij} \) be the matrix with zero entries except for a '1' in the i-th row and j-th column.
$$\begin{align}
+
\hat f(k) &= \frac{1}{2\pi} \int_0^{2\pi} f(x) \: e^{-ikx} \: dx\\
+
&= \frac{1}{2\pi} \left(
+
\frac{\pi^2}{3} \int_0^{2\pi} e^{-ikx} \: dx
+
- \pi \int_0^{2\pi} x \: e^{-ikx} \: dx
+
+ \frac{1}{2} \int_0^{2\pi} x^2 \: e^{-ikx}
+
\right) \\
+
&= \begin{cases}
+
0 & \text{for } k = 0\\
+
\frac{1}{k^2} & \text{for } k \neq 0
+
\end{cases}
+
\end{align}$$
+
which proves that \(B_2(x) = f(x)\) for \(x \in [0, 2\pi]\).
+
  
Thus, we have shown that the \(B_n|_{[0, 2\pi]}\) are polynomials, which are recursively defined by \(B'_n = iB_{n-1}\) and \(\int_0^{2\pi} B_n(x) dx = -i(B_{n+1}(2\pi) - B_{n+1}(0)) = 0\). We also add
+
\(\lambda_i E_{ij} = ME_{ij} = E_{ij}M = \lambda_j E_{ij} \Rightarrow \lambda_i = \lambda_j \)
$$B_1(x) = -iB'_2(x) = i\pi - ix \qquad B_0(x) = -iB'_1(x) = -1$$
+
  
<hr style="border:0;border-top:thin dashed #ccc;background:none;"/>
+
Because \( 1 \leq i \leq j \leq d \) can be chosen arbitrary, this proves the other inclusion \(M \in \{\lambda\mathbb{I}\}\).
  
By induction over \(n\), we will now prove that
+
== Problem 5 (Musterloesung Illmanen) ==
$$B_n(x) = \sum_{k=0}^n B_{n-k}(0) \frac{(ix)^k}{k!}$$
+
Let \(E_{ij} \in \mathbb{K}^{n\times n} \) be the matrix which has the entry 1 at \((i,j)\) and 0 else. From proposition \(AE_{ij} = E_{ij}A ~~ \forall 1 \leq i, j \leq n\). Let \(1 \leq i, j \leq n \) be fixed: \(AE_{ij} \) consists solely out of zeros except for the j-th column which is equal to \((a_{1i}, \ldots , a_{ni})^T\). So we get \( a_{ii} = a_{jj} ~~ \wedge  ~~ a_{ki} = 0 ~~\forall k \neq j\)
Base case for \(n=0\):
+
$$B_0(0) \frac{(ix)^0}{0!} = B_0(0) =  -1 = B_0(x)$$
+
Inductive step (\(n\!-\!1 \rightarrow n\)):
+
$$\begin{align}
+
B_n(x) &= \int_0^x iB_{n-1}(\tilde x)\ d\tilde x + B_n(0) \\
+
&= \int_0^x i\left(\sum_{k=0}^{n-1} B_{(n-1)-k}(0) \frac{(i\tilde x)^k}{k!} \right) d\tilde x + B_n(0) \\
+
&= \left( \sum_{k=0}^{n-1} B_{(n-1)-k}(0) \frac{i^{k+1}}{k!} \int_0^x \tilde x^k\ d\tilde x \right) + B_n(0) \\
+
&= \left( \sum_{k=0}^{n-1} B_{(n-1)-k}(0) \frac{i^{k+1}}{k!} \frac{x^{k+1}}{k+1} \right) + B_n(0) \\
+
&= \left( \sum_{k=0}^{n-1} B_{n-(k+1)}(0) \frac{(ix)^{k+1}}{(k+1)!} \right) + B_n(0) \\
+
&= \left( \sum_{k=1}^n B_{n-k}(0) \frac{(ix)^k}{k!} \right) + B_n(0) \\
+
&= \sum_{k=0}^n B_{n-k}(0) \frac{(ix)^k}{k!}
+
\end{align}$$
+
  
<hr style="border:0;border-top:thin dashed #ccc;background:none;"/>
+
We can do this for all indices and this leads to \(a_{ij} = 0 \forall i \neq j ~~\wedge ~~ a_{11} = a_{22} = \ldots = a_{nn} \). Therefor \(A\) has to have the form \(\lambda E_n, \lambda \in \mathbb{K}\)
  
As \(B_n\) is \(2\pi\)-periodic for all \(n \geq 2\), the above identity leads us to the relation
+
===Ilmanen's solution componentwise===
$$0 = B_n(2\pi) - B_n(0) = \sum_{k=1}^n B_{n-k}(0) \frac{(2\pi i)^k}{k!} \tag{*}$$
+
for all \(n \geq 2\), which determines all \(B_n(0)\) from \(B_0(0) = -1\)
+
  
<hr style="border:0;border-top:thin dashed #ccc;background:none;"/>
+
Let \(A \in Z(\mathrm{Mat}_d(\mathbb{C}))\).
  
The definition of the \(B_n\) in the problem (the Bernoulli numbers, not the functions!) can be transformed to:
+
Let \(E_{ij}\) be the \(d \times d\)-matrix with \((E_{ij})_{kl} = \delta_{ik} \delta_{jl} \).
$$\begin{align}
+
& \frac{z}{e^z - 1} = \sum_{n=0}^\infty B_n \frac{z^n}{n!} \\
+
\Leftrightarrow\ & z = \left( \sum_{n=0}^\infty B_n \frac{z^n}{n!} \right) \left( \sum_{k=1}^\infty \frac{z^k}{k!} \right) \\
+
\Leftrightarrow\ & z = \left( B_0 + B_1z + \frac{B_2}{2}z^2 + \frac{B_3}{3!}z^3 + ... \right) \left( z + \frac{z^2}{2} + \frac{z^3}{3!} + \frac{z^4}{4!} + ... \right) \\
+
\Leftrightarrow\ & z = B_0z + \left( \frac{B_0}{2!} + B_1 \right)z^2 + \left( \frac{B_0}{3!} + \frac{B_1}{2!} + \frac{B_2}{2!} \right)z^3 + \left( \frac{B_0}{4!} + \frac{B_1}{3!} + \frac{B_2}{2!2!} + \frac{B_3}{3!} \right)z^4
+
\end{align}$$
+
which, through equating coefficients yields \(B_0 = 1\) and for \(n \geq 2\)
+
$$\sum_{k=1}^n \frac{B_{n-k}}{(n-k)!k!} = 0 \tag{$\dagger$}$$
+
  
<hr style="border:0;border-top:thin dashed #ccc;background:none;"/>
+
Now let \(1 \leq i, j \leq d\) with \(i \neq j\) be fixed and consider
 +
$$(E_{ij} A)_{kl} = \sum \limits_{m}(E_{ij})_{km} A_{ml} = \sum \limits_{m}\delta_{ki} \delta_{jm} A_{ml} = \delta_{ki} A_{jl}.$$
 +
Since \(A\) commutes with all compex \(d \times d\)-matrices, this is the same as
 +
$$(A E_{ij})_{kl} = \sum \limits_{m}A_{km} (E_{ij})_{ml} = \sum \limits_{m} A_{km} \delta_{mi} \delta_{jl} = \delta_{jl} A_{ki}.$$
  
<span style="color:#339;font-size:80%;">The following proof by induction doesn't appear in the aforementioned Musterlösung, where the final step simply falls out of thin air. It is not unlikely, however, that I simply don't understand what they do. If this is the case, feel free to simplify the following mess of equations. --Nik</span>
+
Thus, we have that
 
+
$$\delta_{ki} A_{jl} = \delta_{jl} A_{ki}$$
We now now prove by induction that for \(n \geq 0\)
+
Now set \(k=i, l=j\). We then find
$$B_n(0) = - \frac{(2\pi i)^n B_n}{n!}$$
+
$$A_{ii} = A_{jj}$$
 
+
Next, set \(k=l=i\). Still, \(j \neq i\). It follows that
Base case for \(n=0\):
+
$$A_{ji} = 0.$$
$$B_0(0) = -1 = -B_0$$
+
As this holds for an arbitrary choice of \(i \neq j\), the required form for \(A\) is
Inductive step (\(n\!-\!1 \rightarrow n\)):
+
$$A = \lambda \mathbb{I}_d$$
$$\begin{align}
+
for some \(\lambda \in \mathbb{C}\). And obviously \(\forall \lambda \in \mathbb{C}: \lambda \mathbb{I} \in Z(\mathrm{Mat}_d(\mathbb{C}))\).
\sum_{k=1}^{n+1} B_{n+1-k}(0) \frac{(2\pi i)^k}{k!} \ &\overset{(*)}{=} \ 0 \\
+
\Leftrightarrow \ B_n(0)\: 2\pi i \ &= \: - \sum_{k=2}^{n+1} B_{n+1-k}(0) \frac{(2\pi i)^k}{k!} \\
+
\overset{\text{Hyp.}}{\Leftrightarrow} \ B_n(0)\: 2\pi i \ &= \: - \sum_{k=2}^{n+1} \left( - \frac{(2\pi i)^{n+1-k} B_{n+1-k}}{(n+1-k)!} \right) \frac{(2\pi i)^k}{k!} \\
+
\Leftrightarrow \ B_n(0)\: 2\pi i \ &= \ (2\pi i)^{n+1} \sum_{k=2}^{n+1} \frac{B_{n+1-k}}{(n+1-k)!\:k!} \\
+
\overset{(\dagger)}{\Leftrightarrow} \ B_n(0)\: 2\pi i \ &= \ (2\pi i)^{n+1} \left(- \frac{B_n}{n!} \right) \\
+
B_n(0) \ &= \: - \frac{(2\pi i)^n B_n}{n!}
+
\end{align}$$
+
 
+
This, together with equation \((\ddagger)\), proves the identity in the problem.
+
 
+
<p style="text-align:right;">\(\square\)</p>
+
 
+
 
+
% =Alternate Solution=
+
% Let $\Zeta(s) = \sum\limits_{n=0}^\infty \frac{1}{n^s}$ for $\left{s \in \mathbb{C}\mid \Re (s) > 1 \right}$ \\
+
% Consider
+
% $$ E(z) = \frac{z}{e^z-1} = \sum\limits_{n=0}^\infty B_n \frac{z^n}{n!} \quad\quad\quad z \in \mathbb(R)$$
+
 
+
 
+
% =Alternate Solution=
+
 
+
Let $$\zeta(s) = \sum\limits_{n=1}^\infty \frac{1}{n^s} $$ for $$s \in \mathbb{C}, \Re (s) > 1$$
+
 
+
Consider
+
$$ E(z) := \frac{z}{e^z -1} := \sum\limits_{n=0}^\infty B_n \frac{z^n}{n!} \quad\quad\quad z \in \mathbb{R} $$
+
$$\lim\limits_{z \to 0}{E(z)} = \lim\limits_{z \to 0}{\frac{1}{e^z}} = 1 = \lim\limits_{z \to 0}{\sum\limits_{n=0}^\infty B_n \frac{z^n}{n!}} $$
+
Where in the first equality we used l'Hospital's rule and with
+
$$ \lim\limits_{z \to 0}{E(z)} = E(\lim\limits_{z \to 0}{z}) = \sum\limits_{n=0}^\infty \lim\limits_{z \to 0}{B_n \frac{z^n}{n!}} =B_0 $$
+
Where we can put the limit inside because E is continuous and bounded in a neighborhood of $$z_0 = 0$$.\\
+
$$ \rightarrow B_0 = 1 $$
+
Now we repeat the same thing for $$E'$$ \\
+
\begin{align*}
+
\lim\limits_{z \to 0}{E'(z)} & = \lim\limits_{z \to 0}{\frac{e^z -1 -z e^z}{e^{2z} -2e^z+1}} \\
+
& = \lim\limits_{z \to 0}{\frac{e^z -z e^z -e^z}{2e^{2z} -2e^z}} \\
+
& = \lim\limits_{z \to 0}{\frac{-z e^z - e^z}{4e^{2z}-2e^z}} \\
+
& = -\frac{1}{2}
+
\end{align*}
+
Here we again used l'Hospital's rule for the second and third equality. For the same reasons as above we get
+
$$ \lim\limits_{z \to 0}{E'(z)} = B_1 $$
+
and therefore
+
$$\rightarrow b_1 = - \frac{1}{2}$$
+
 
+
$$\Rightarrow E(z) = 1 - \frac{1}{2} z + \mathcal{O}(z^2)$$
+
 
+
Now define $$f(z)  = \frac{z}{2} \coth (\frac{z}{2})$$ on $$]-\frac{1}{2}, \frac{1}{2}]$$\\
+
\begin{align*}
+
f(z) & = \frac{z}{2} \coth (\frac{z}{2}) \\
+
& = \frac{z e^{\frac{z}{2}} + z e^{-\frac{z}{2}}}{2e^{\frac{z}{2}} - 2 e^{-\frac{z}{2}}} \\
+
& = \frac{z e^{\frac{z}{2}} + z e^{-\frac{z}{2}}}{2e^{\frac{z}{2}}(e^z-1)} \\
+
& = \frac{z}{2} \frac{e^z+1}{e^z-1} \\
+
& = \frac{z}{2} \left( 1 + \frac{2}{e^z-1} \right) \\
+
& = \frac{z}{2} + \frac{z}{e^z-1} \\
+
& = \frac{z}{2} + E(z)
+
\end{align*}
+
 
+
Since $$z$$ and $$\coth$$ are odd functions, $$f(z)$$ is an even function. From the part above we know that $$E(z) = 1 - \frac{1}{2} z + \mathcal{O}(z^2)$$. Since the addition of $$\frac{z}{2}$$ kills the uneven term and $$f(z)$$ has to be even, the only remaining coefficients are even:\\
+
$$ f(z) = \frac{z}{2} + E(z) = \sum\limits_{n=0}^\infty B_{2n} \frac{z^{2n}}{(2n)!} $$
+
 
+
Now define $$g(x) = \cosh (zx)$$ on $$]-\frac{1}{2}, \frac{1}{2}]$$
+
with $$z \in \mathbb{R} \setminus \{0\}$$, continued periodically on $$\mathbb{R}$$. \\
+
Calculating the Fourier coefficients of $$g$$: \\
+
 
+
$$\alpha_0 = \int\limits_{-\frac{1}{2}} ^{\frac{1}{2}} \cosh(zx)dx = \frac{2}{z}\sinh (\frac{z}{2})$$
+
 
+
\begin{align*}
+
\alpha_n & = \int\limits_{-\frac{1}{2}} ^{\frac{1}{2}} \cosh(zx)\cos(2\pi n x)dx \\
+
& = \left[\cosh(zx)\frac{1}{2\pi n}\sin(2\pi nx) \right]_{-\frac{1}{2}}^{\frac{1}{2}} - \int\limits_{-\frac{1}{2}} ^{\frac{1}{2}} z \sinh (zx) \frac{1}{2\pi n} \sin (2\pi nx) dx \\
+
& = -z \int\limits_{-\frac{1}{2}} ^{\frac{1}{2}}\sinh (zx) \frac{1}{2\pi n} \sin (2\pi nx) dx \\
+
& = -z \left[ \left[-\sinh (zx) \left(\frac{1}{2\pi n}\right)^2 \cos (2\pi nx) \right]_{-\frac{1}{2}}^{\frac{1}{2}} + z \int\limits_{-\frac{1}{2}}^{\frac{1}{2}} \cosh (zx)\left(\frac{1}{2\pi n}\right)^2 \cos (2\pi nx) dx \right]
+
\end{align*}
+
 
+
$$ \Rightarrow \left(1 + \left(\frac{z}{2\pi n}\right)^2\right)\int\limits_{-\frac{1}{2}}^{\frac{1}{2}} \cosh (zx) \cos (2\pi nx) dx = 2z\left(\frac{z}{2\pi n}\right)^2 (-1)^n \sinh (\frac{z}{2}) $$
+
 
+
$$ \rightarrow \alpha_n = \int\limits_{-\frac{1}{2}}^{\frac{1}{2}} \cosh (zx) \cos (2\pi nx) dx = \frac{2z}{(2\pi n)^2 + z^2} (-1)^n \sinh (\frac{z}{2}) $$
+
Where we use $$\cos(2\pi n x)$$ because the integrand is even. Therefore we get for the Fourieseries of $$g$$:\\
+
 
+
$$ \cosh (zx)  = \frac{2}{z} \sinh (\frac{z}{2}) \left[ 1 + \sum\limits_{n \in \mathbb{Z} \\ n \neq 0} (-1)^n \frac{2z}{4{\pi}^2 n^2 + z^2} \cos (2\pi n x) \right] $$
+
$$ = \frac{2}{z} \sinh(\frac{z}{2}) \left[ 1 + 2\sum\limits_{n=1}^\infty (-1)^n \frac{2z}{4{\pi}^2 n^2 + z^2} \cos (2\pi n x) \right]$$
+
 
+
Now we chose $$x = \frac{1}{2}$$:
+
 
+
$$ \cosh (\frac{z}{2}) = \frac{2}{z} \sinh(\frac{z}{2}) \left[ 1 + 2\sum\limits_{n=1}^\infty (-1)^n \frac{z^2}{4\pi^2 n^2} \frac{1}{1+\frac{z^2}{4\pi^2 n^2}} \right] $$
+
 
+
$$ \underbrace{\frac{z}{2} \frac{\cosh (\frac{z}{2})}{\sinh(\frac{z}{2})}}_{f(z)}
+
= 1 + 2\sum\limits_{n=1}^\infty (-1)^n \frac{z^2}{4{\pi}^2 n^2} \frac{1}{1+\frac{z^2}{4{\pi}^2 n^2}} $$
+
$$ = 1 + \sum\limits_{n=1}^\infty \sum\limits_{k=0}^\infty \frac{2z^2}{4{\pi}^2 n^2} \left(-\frac{z^2}{4\pi^2 n^2}\right)^k$$
+
 
+
Because both series are absolute convergent and uniform convergent we get after some calculation:
+
$$ f(z) = 1 + \sum\limits_{k = 1}^\infty 2 (-1)^{k-1} \zeta(2k) \frac{z^{2k}}{(2\pi)^{2k}} $$
+
but above we also calculated
+
 
+
$$ f(z) =1 +  \sum\limits_{n=1}^\infty B_{2n} \frac{z^{2n}}{(2n)!} $$
+
With a coefficient comparison we get:
+
 
+
$$ \zeta(2n) = - \frac{B_{2n}(-1)^{2n}(2\pi)^{2n}}{2(2n)!} = -\frac{(2\pi i)^{2n}}{2(2n)!}B_{2n} \quad\quad\quad \forall n \in \mathbb{N} $$
+
which is what we had to show.
+

Latest revision as of 14:01, 30 July 2015

Follow the link! (Original Solution Sheldon) [1]

An alternative solution can be found here (3.). (MuLö Linalg) [2]

Task

Determine the center of the algebra of complex d × d-matrices. Hint: the center of an algebra \(A\) is defined as \(Z(A) = \{\phi \in A | \phi \psi = \psi \phi \; \forall \psi \in A \} \).

Problem 5 (Sheldon)

Let big bold letters denote elements of \(\mathbb{C}^{d \times d}\) and small Greek letters complex numbers.

Let \(M \in Z\)

We make the following observations and conclude that \( Z = \{\lambda\mathbb{I}\}\)


Obviously, the elementes of the center are closed under scalar multiplication (they even form a vector space over \(\mathbb{C}\)). \(\mathbb{I} \in Z \Rightarrow \{\lambda\mathbb{I}\} \subseteq Z\)


The theorem about Jordan decomposition states that \(M\) is conjugate to a matrix in Jordan normal form \(J\) and \(J = CMC^{-1}=MCC^{-1}=M\).

\(M\) is a diagonal matrix.

Suppose to the contrary that \(M\) has a '1' in the superdiagonal, then \(2M\) is supposed to be in the center as well, but it can't, because it is not in Jordan normal form.


All diagonal entries of \(M\) are equal.

Let \( M =\begin{pmatrix} \lambda_1 & 0 & \ldots & 0 \\ 0 & \lambda_2 & \ldots & 0 \\ \vdots & &\ddots& \\ 0 & \ldots & 0 & \lambda_d \end{pmatrix} \) and \( E_{ij} \) be the matrix with zero entries except for a '1' in the i-th row and j-th column.

\(\lambda_i E_{ij} = ME_{ij} = E_{ij}M = \lambda_j E_{ij} \Rightarrow \lambda_i = \lambda_j \)

Because \( 1 \leq i \leq j \leq d \) can be chosen arbitrary, this proves the other inclusion \(M \in \{\lambda\mathbb{I}\}\).

Problem 5 (Musterloesung Illmanen)

Let \(E_{ij} \in \mathbb{K}^{n\times n} \) be the matrix which has the entry 1 at \((i,j)\) and 0 else. From proposition \(AE_{ij} = E_{ij}A ~~ \forall 1 \leq i, j \leq n\). Let \(1 \leq i, j \leq n \) be fixed: \(AE_{ij} \) consists solely out of zeros except for the j-th column which is equal to \((a_{1i}, \ldots , a_{ni})^T\). So we get \( a_{ii} = a_{jj} ~~ \wedge ~~ a_{ki} = 0 ~~\forall k \neq j\)

We can do this for all indices and this leads to \(a_{ij} = 0 \forall i \neq j ~~\wedge ~~ a_{11} = a_{22} = \ldots = a_{nn} \). Therefor \(A\) has to have the form \(\lambda E_n, \lambda \in \mathbb{K}\)

Ilmanen's solution componentwise

Let \(A \in Z(\mathrm{Mat}_d(\mathbb{C}))\).

Let \(E_{ij}\) be the \(d \times d\)-matrix with \((E_{ij})_{kl} = \delta_{ik} \delta_{jl} \).

Now let \(1 \leq i, j \leq d\) with \(i \neq j\) be fixed and consider $$(E_{ij} A)_{kl} = \sum \limits_{m}(E_{ij})_{km} A_{ml} = \sum \limits_{m}\delta_{ki} \delta_{jm} A_{ml} = \delta_{ki} A_{jl}.$$ Since \(A\) commutes with all compex \(d \times d\)-matrices, this is the same as $$(A E_{ij})_{kl} = \sum \limits_{m}A_{km} (E_{ij})_{ml} = \sum \limits_{m} A_{km} \delta_{mi} \delta_{jl} = \delta_{jl} A_{ki}.$$

Thus, we have that $$\delta_{ki} A_{jl} = \delta_{jl} A_{ki}$$ Now set \(k=i, l=j\). We then find $$A_{ii} = A_{jj}$$ Next, set \(k=l=i\). Still, \(j \neq i\). It follows that $$A_{ji} = 0.$$ As this holds for an arbitrary choice of \(i \neq j\), the required form for \(A\) is $$A = \lambda \mathbb{I}_d$$ for some \(\lambda \in \mathbb{C}\). And obviously \(\forall \lambda \in \mathbb{C}: \lambda \mathbb{I} \in Z(\mathrm{Mat}_d(\mathbb{C}))\).